Menu Close

Given-p-q-R-1-1-p-1-q-1-show-that-a-b-R-ab-a-p-p-b-q-q-




Question Number 97637 by Ar Brandon last updated on 09/Jun/20
Given p,q∈R_+ ^∗ −{−1}/(1/p)+(1/q)=1 show that;  ∀a,b ∈R ab≤(a^p /p)+(b^q /q)
$$\mathrm{Given}\:\mathrm{p},\mathrm{q}\in\mathbb{R}_{+} ^{\ast} −\left\{−\mathrm{1}\right\}/\frac{\mathrm{1}}{\mathrm{p}}+\frac{\mathrm{1}}{\mathrm{q}}=\mathrm{1}\:\mathrm{show}\:\mathrm{that}; \\ $$$$\forall\mathrm{a},\mathrm{b}\:\in\mathbb{R}\:\mathrm{ab}\leqslant\frac{\mathrm{a}^{\mathrm{p}} }{\mathrm{p}}+\frac{\mathrm{b}^{\mathrm{q}} }{\mathrm{q}} \\ $$
Commented by arcana last updated on 09/Jun/20
its the Young′s inequality. You can  find it in various web pages
$$\mathrm{its}\:\mathrm{the}\:\mathrm{Young}'\mathrm{s}\:\mathrm{inequality}.\:\mathrm{You}\:\mathrm{can} \\ $$$$\mathrm{find}\:\mathrm{it}\:\mathrm{in}\:\mathrm{various}\:\mathrm{web}\:\mathrm{pages} \\ $$

Leave a Reply

Your email address will not be published. Required fields are marked *